K
Khách

Hãy nhập câu hỏi của bạn vào đây, nếu là tài khoản VIP, bạn sẽ được ưu tiên trả lời.

20 tháng 11 2017

đặt A=...

Áp dúng bất đẳng thức bu nhi a ta có 

\(A^2\le3\left(1+a^2+2bc+1+b^2+2ac+1+c^2+2ab\right)=3\left[\left(a+b+c\right)^2+3\right]\)

=> \(A^2\le36\Rightarrow A\le6\) (ĐPCM)

dấu = xảy ra <=> a=b=c=1

26 tháng 8 2015

Theo bất đẳng thức Cauchy-Schwartz ta có

\(\frac{1}{a^2+2bc}+\frac{1}{b^2+2ca}+\frac{1}{c^2+2ab}\ge\frac{\left(1+1+1\right)^2}{a^2+2bc+b^2+2ca+c^2+2ab}=\frac{9}{\left(a+b+c\right)^2}\ge\frac{9}{1}=9.\)

a2+b2+c2=(a+b+c)2<=> ab+bc+ca=0

\(\Rightarrow S=\frac{a^2}{a^2+bc-\left(ab+ca\right)}+\frac{b^2}{b^2+ac-\left(ab+bc\right)}+\frac{c^2}{c^2+ab-\left(bc+ca\right)}\)

\(=\frac{a^2}{\left(a-b\right)\left(a-c\right)}-\frac{b^2}{\left(b-c\right)\left(a-b\right)}-\frac{c^2}{\left(b-c\right)\left(c-a\right)}\)

\(=\frac{a^2\left(b-c\right)-b^2\left(a-c\right)-c^2\left(a-b\right)}{\left(a-b\right)\left(b-c\right)\left(c-a\right)}=\frac{\left(a-b\right)\left(b-c\right)\left(c-a\right)}{\left(a-b\right)\left(b-c\right)\left(c-a\right)}=1\)

M  tương tự

7 tháng 1 2018

\(VT\ge a+b+c+\dfrac{9}{2\left(ab+bc+ca\right)}\ge\sqrt{3\left(ab+bc+ca\right)}+\dfrac{9}{2\left(ab+bc+ca\right)}\)

\(=\dfrac{\sqrt{3\left(ab+bc+ca\right)}}{2}+\dfrac{\sqrt{3\left(ab+bc+ca\right)}}{2}+\dfrac{9}{2\left(ab+bc+ca\right)}\ge3\sqrt[3]{\dfrac{27}{8}}=\dfrac{9}{2}\)

7 tháng 1 2018

Áp dụng BĐT Cauchy ta có

\(\dfrac{b^2}{a}+a\ge2b;\) \(\dfrac{c^2}{b}+b\ge2c\); \(\dfrac{a^2}{c}+c\ge2a\)

\(\Rightarrow\dfrac{b^2}{a}+\dfrac{c^2}{b}+\dfrac{a^2}{c}\ge a+b+c\)

\(\Rightarrow\dfrac{b^2}{a}+\dfrac{c^2}{b}+\dfrac{a^2}{c}+\dfrac{9}{2\left(ab+bc+ac\right)}\ge a+b+c+\dfrac{9}{2\left(ab+bc+ac\right)}\)Ta phải chứng minh

\(a+b+c+\dfrac{9}{2\left(ab+bc+ac\right)}\ge\dfrac{9}{2}\)

\(\Leftrightarrow4\left(a+b+c\right)\left(ab+bc+ac\right)+18\ge18\left(ab+bc+ac\right)\)

\(\Leftrightarrow\left(ab+bc+ac\right)\left(4\left(a+b+c\right)-18\right)+18\ge0\)

Áp dụng BĐT Cauchy:

\(ab+bc+ac\ge3\sqrt[3]{a^2b^2c^2}=3\)

\(a+b+c\ge3\sqrt[3]{abc}=3\)

\(\Rightarrow\left(ab+bc+ac\right)\left(4\left(a+b+c\right)-18\right)+18\ge3\left(4.3-18\right)+18=0\)=> đpcm

24 tháng 11 2017

Áp dụng BĐT Cauchy Swarch

\(\Sigma\dfrac{1}{a^2+2bc}\ge\dfrac{9}{\left(a+b+c\right)^2}=9\)

Vậy Min ... =9 khi a=b=c=1/3

15 tháng 5 2016
Dùng Svaxơ là ra nha bạn
16 tháng 10 2018

Áp dụng BĐT Cauchy - Schwarz vào bài toán , ta có :

\(Q=\dfrac{1}{a^2+2bc}+\dfrac{1}{b^2+2ac}+\dfrac{1}{c^2+2ab}\ge\dfrac{\left(1+1+1\right)^2}{a^2+b^2+c^2+2ab+2bc+2ac}=\dfrac{9}{\left(a+b+c\right)^2}=\dfrac{9}{1^2}=9\) Dấu " = " xảy ra khi : \(\dfrac{1}{a^2+2ab}=\dfrac{1}{b^2+2ac}=\dfrac{1}{c^2+2ab}\Leftrightarrow a=b=c=\dfrac{1}{3}\)

\(\Rightarrow Q_{Min}=9\Leftrightarrow a=b=c=\dfrac{1}{3}\)